Fall 2010 Final CC-BY-NC

Maintainer: admin

Student-provided questions and solutions to the Fall 2010 final exam.

It is recommended that you do this exam yourself before checking the solutions provided here. If you notice any errors with the content on this page, feel free to either contact @dellsystem or edit the page directly.

1Question 1

(i) The Archimedean property states that the set of natural numbers $\mathbb N$ is unbounded in $\mathbb R$. Assuming this property, show that $\inf \{1/n: n \in \mathbb N\} = 0$. Conclude that if $t > 0$, there exists some $n \in \mathbb N$ with $\frac{1}{n} < t$, and if $y > 0$, there exists a natural number $n$ such that $n-1 \leq y < n$.
(ii) Use (i) to show that for any two real numbers $x$ and $y$ with $x < y$, there exists a rational number $r$ with $x < y < y$.

1.1Solution

Coming soon

1.2Accuracy and discussion

2Question 2

(i) Let $f : A \to \mathbb R$ be a function. Define what it means for $f$ to be continuous on $A$, and uniformly continuous on $A$. Comment on the difference between these definitions.
(ii) Show that if $f$ and $g$ are uniformly continuous functions on $A$, and they are both bounded on $A$, then their product $fg$ is uniformly continuous on $A$.
(iii) Show that if $A$ is the closed bounded interval $A = [a, b]$, then a continuous function $f: A \to \mathbb R$ must be bounded. State any theorems you use. Conclude that the product of two uniformly continuous functions defined on a closed bounded interval is uniformly continuous.
(iv) Give an example of a set $A$ and a uniformly continuous function $f: A \to \mathbb R$ which is not bounded, justifying your choice.

2.1Solution

Coming soon

2.2Accuracy and discussion

3Question 3

(i) Let $a \subset \mathbb R$ be a nonempty set. Define what it means for a real number to be the supremum of $A$. State the completeness axiom for $\mathbb R$.

Now let $I_n = [a_n, b_n]$, $n\in \mathbb N$ be a nested sequence of intervals, i.e., $I_{n+1} \subset I_n$ for each $n \in \mathbb N$.

(ii) Show that the set $\{a_n:n \in \mathbb N\}$ is bounded above, and let $a*$ be its supremum.
(iii) Show that $\displaystyle a* \in \cap_{n=1}^{\infty} I_n$.
(iv) If $f: [a, b] \to \mathbb R$ is a continuous function, with $f(a) < 0$ and $f(b) > 0$, show that there exists $x \in (a, b)$ with $f(x) = 0$. Hint: use part (iii).
(v) Show that the polynomial $f(x) = x^4 + 7x^3 - 9$ has at least 2 real roots. Describe how you would locate one of these roots with error less than 0.005.

3.1Solution

Coming soon

3.2Accuracy and discussion

4Question 4

Prove or disprove each of the following statements.

(i) If every subsequence of $(x_n)$ has a subsequence that converges to 0, then $\displaystyle \lim_{n \to \infty} x_n = 0$.
(ii) The sequence $(1 + (-1)^n)$ is a Cauchy sequence.
(iii) The sequence $((3n)^{\frac{1}{2n}})$ diverges to $\infty$.

4.1Solution

(i) I have no idea. That's interesting.
(ii) Disprove.
(iii) Disprove. It converges to 1 for some reason. Why???

4.2Accuracy and discussion

5Question 5

(i) If $0 < x < 1$, show that $x^n \to 0$ as $n \to \infty$.
(ii) If $0 < a <b$, find the limit of the sequence $\displaystyle \left ( \frac{a^{n+1} + b^{n+1}}{a^n + b^n} \right )$, stating any theorems that you use.

5.1Solution

(ii) This is straight out of the textbook. Coming soon.

5.2Accuracy and discussion

6Question 6

(i) Suppose $f: [a, b] \to \mathbb R$ with $c \in (a, b)$. Show that $f$ is differentiable at $c$ if and only if there exists a function $\phi: [a, b] \to \mathbb R$ such that $\mathbb \phi$ is continuous at $c$ and satisfies

$$f(x) - f(c) = \phi(x)(x-c)$$

whenever $x \in [a, b]$. Show that in this case, $f'(c) = \phi(c)$.
(ii) If $f(x) = x^3$, find $\phi(x)$ and deduce that $f'(c) = 3c^2$.
(iii) Suppose $f$ is strictly monotone increasing and continuous on $[a, b]$. Let $J = f([a, b])$ and consider the function $g: J \to [a, b]$ defined as $g = f^{-1}$. Explain why $J$ is a closed bounded interval. Show that if $f$ is differentiable at $c \in [a, b]$ and $f'(c) \neq 0$, then $f^{-1}$ is differentiable at $d = f(c)$ and $g'(d) = \frac{1}{f'(c)}$. Hint: use part (i). Then, using part (ii), if $f(x) = x^3$, find $g'(d)$.

6.1Solution

Coming soon

6.2Accuracy and discussion

7Question 7

Let $f$, $g$ be differentiable on $\mathbb R$ and suppose that $f(0) = g(0)$ and $f'(x) \leq g'(x)$ for all $x \geq 0$. Show that $f(x) \leq g(x)$ for all $x \geq 0$. State completely any theorems that you use.

7.1Solution

Coming soon

7.2Accuracy and discussion